If the table is auctioned on a date that is later than both the date on which the mirror is auctioned and the date on...

RyanSpencer on September 12, 2019

How is A wrong?

how is A incorrect? Wouldn't L,S,H,T,M,V work? That would put H>T

Replies
Create a free account to read and take part in forum discussions.

Already have an account? log in

Skylar on September 13, 2019

@RyanSpencer Happy to help.

The question states "If the table is auctioned on a date that is later than both the date on which the mirror is auctioned and the date on which the vase is auctioned, then which one of the following could be true?" This means that we must put M>T and V>T. In your suggestion of L-S-H-T-M-V, this condition is not met as T is before both M and V.

I hope this helps to clarify. Please let us know if you have any additional questions!

RyanSpencer on September 13, 2019

Oh. Ok, I understand now. I misread the question. Thank you!